Open query: causes of denial and delay

I have my own theories about why people either:

Why do readers of this blog believe that both stances are so prevalent, at a time when the key science behind climate change is well understood?

Author: Milan

In the spring of 2005, I graduated from the University of British Columbia with a degree in International Relations and a general focus in the area of environmental politics. In the fall of 2005, I began reading for an M.Phil in IR at Wadham College, Oxford. Outside school, I am very interested in photography, writing, and the outdoors. I am writing this blog to keep in touch with friends and family around the world, provide a more personal view of graduate student life in Oxford, and pass on some lessons I've learned here.

106 thoughts on “Open query: causes of denial and delay”

  1. I think most people believe climate change is happening, and yet choose to do nothing about it. Ergo, the theory of the subject in terms of desire+belief=action is simply false. Most actions are done not “rationally” but out of peer pressure, or some other complex social force.

  2. I think the rationale (or lack thereof) of people who aren’t in a position to influence change from above goes something like:

    Climate change is a huge and frightening prospect to begin tackling individually. You either choose:

    a. To believe that one’s actions can make a difference, and act out individually, make personal sacrifices feeling that it will contribute to the overall good of society, or

    b. Believe that a single person’s actions are insignificant, and feel that acting out would only make the fleeting life of comfort we are experiencing uncomfortable by making personal sacrifices.

    c. Cannot believe in climate change disaster altogether. Either because they believe that there is a benevolent god who is watching over them and theirs, or it is just a firm faith in human safety (or sheer inability to use their imagination), in spite of scientific evidence.

    —-

    I choose (a) because gosh and golly I have a crazy belief in our ability to tackle this in a meaningful way, and I think that support for that view is growing quickly. (b) I can see as a very attractive option, if not an ethically sound, or responsible one. And (c) I can understand, from the stance of a skeptic who has seen one too many ‘scientifically’ anticipated disease, technology, or killer bee related epidemics talked about to but never manifested in a serious way.

    I think it is important to continually question and re-explore, re-evaluate the data on climate change. As it is has been re-asserted here many a time, there are overwhelming trends in the data being collected, all pointing towards climate changes occurring now and increasing in severity with no change in human behaviour. That doesn’t mean that we shouldn’t be questioning the data, if we have concerns with its reliability.

    It is perhaps encouraging in a way that despite growing pressure on people to simply ‘believe’ in climate change (let’s face it, you look like a hick if you deny it altogether), people continue to demand more factual evidence. Sure it’s exhausting, but the scariest people of all are maybe those who refuse to ask questions altogether about popular beliefs.

    Though, somewhat more depressingly, you can’t convince people of the faith with scientific evidence. It’s like when Satan buried those dinosaur bones in the earth to trick us into believing in Darwinian theories of evolution.

    Your blog is a really wonderful place to generate open discussion on new data and trends relating to climate changes, and I’m glad to be able to follow threads of conflicting opinions and beliefs about it.

    Just to make it clear, I do think that there is enough reliable scientific data pointing towards anthropogenically induced climate changes – and I do think that we should be making a concerted effort to reduce our impact.

    But I do appreciate people who question thoroughly before they ‘believe’.

  3. I think most people believe that climate change is happening and that humans are contributing to it. Also the problem is so significant and global that any one person’s individual attempt to reduce carbon emissions is simply insignificant. This is why the change must be done by governments national and international.

    I think it is a bit like taxes. We recognize we need to pay taxes to pay for government-funded services like schools, hospitals and roads. If the citizens of a community were voluntarily and individually to determine how much taxes they paid, the revenue collected would be minimal . The services would disappear. We agree to pay taxes as determined by government for the greater common good because left to our own individual decisions we would choose not to pay taxes.

  4. I think Emily is ignoring a serious option – believe it exists, believe that individual actions can’t stop it, and work together with others towards the cultural shift that can. Secondary effects are much more powerful than primary ones.

  5. Oleh is right. Stopping climate change by reducing your personal carbon footprint is like trying to expand the welfare state by voluntarily paying higher taxes.

  6. I think many people won’t believe in anything that is inconvenient for them or not in their best interests. If they believed climate change is happening they would have make significant changes in their lifestyles – changes they are not willing to make. So they shut their ears/eyes to the evidence and choose to believe the people who tell them it’s all a bunch of hooey

  7. I honestly do not understand why people deny it is happening. I feel like the science is sound. I’m very interested to hear your own thoughts on why people might outright deny it.

    I liked Emily’s comment — I can definitely see people believing it to be true, but feeling like there is nothing they can do as individuals, and so not bothering to make any changes. But to completely disbelieve? To say it just is not happening? Is so strange.

  8. Hi Milan, long time no see.

    I hate to admit this, but I am actually becoming a skeptic myself. A moderate skeptic — I’m sure that human activity is having an effect, and I believe that any effort to reduce our consumption of fossil fuels is a good one. My skepticism is this: are we really so sure about our science and the doomsday scenarios that it predicts? I have heard from a number (well, 2) reputable sources that climate scientists flat out don’t know how the increase in water vapor in the atmosphere caused by global warming will affect global temperature. It’s either going to increase the greenhouse effect, leading to a positive feedback loop of doom, or shield the Earth from sunlight and lead to a negative, self-regulating feedback loop. Forgive me if I have misstated any facts and feel free to correct me on that last bit.

    So it’s possible that doom is just around the corner, or we have nothing to worry about. And honestly, given that the Earth has probably gone through periods of warming before, it wouldn’t be surprising at all if there was a self-regulation mechanism.

    The other reason I have become skeptical is due to my first-hand experience with academic research. There is a tremendous incentive to produce research with the “correct” results (the results everybody wants to hear). While the majority of people in my field appear to me to be totally honest and well-meaning, papers are often published where every effort is made to support a desirable conclusion without outright dishonesty. For climate change scientists, their field would lose tremendous amounts of funding if they ever came to the conclusion that climate change wasn’t anything to worry about. So every individual actor makes sure to come up with the “right” results, because they know if they don’t, somebody else will, and funding will be appropriately re-allocated.

    Again, I would like to re-iterate that I am not doubting the integrity of researchers. I firmly believe that climate change scientists belief their conclusions, and their is a good deal of evidence to support the claims. But I would not at all be surprised if evidence that casts doubt on the catastrophic predictions of global warming is somewhat ignored in the continual quest to publish and justify the continued funding of their research.

    I am writing this because I want to be convinced otherwise. I don’t want to be a skeptic, and want to hear from someone who I believe to be knowledgeable in the subject to convince me otherwise. How sure are you that anthropogenic climate change is really the doomsday scenario we are led to believe?

  9. I have heard from a number (well, 2) reputable sources that climate scientists flat out don’t know how the increase in water vapor in the atmosphere caused by global warming will affect global temperature. It’s either going to increase the greenhouse effect, leading to a positive feedback loop of doom, or shield the Earth from sunlight and lead to a negative, self-regulating feedback loop.

    This is not something I have ever heard. The amount of water vapour in the atmosphere is determined by the temperature, and the more water vapour there is, the more outgoing infrared radiation is absorbed instead of escaping into space. As such, water vapour in the atmosphere is definitely a positive feedback. If you want a lot more detail, as well as an account of the history of climate science, Spencer Weart has an excellent book available free online.

    While there may be some negative feedbacks (such as plants growing more in a higher CO2 environment), there are probably more positive feedbacks (albedo decrease from melting ice, methane from permafrost, forests drying and burning, etc).

    One major source of uncertainty about how much temperature change we will see is what the future course of human emissions will be. The difference between a business-as-usual scenario and one with strong policy is vast. That form of uncertainty accounts for more than half of the uncertainty in IPCC estimates.

    There is a hierarchy of uncertainty, when it comes to climate change. Exactly how bad it will be is the hardest thing to know (since we don’t have other planets to experiment on). What I think is clearly the case is that there is a credible risk that warrants precautionary action.

  10. And honestly, given that the Earth has probably gone through periods of warming before, it wouldn’t be surprising at all if there was a self-regulation mechanism.

    First, we know from paleoclimatic records that there have been abrupt lunges in climate before, where a small initial change was magnified by positive feedbacks. Indeed, this is the mechanism through which small orbital changes cause major climatic changes over very long timescales.

    Secondly, while both the Earth and life are safe enough, there is good reason to question how much change humanity could endure. There are seven billion of us and counting, living in societies that are adapted to the climate as it exists now. If we start to see major rivers drying up, reducing agricultural production significantly, we could soon find ourselves in quite a tight spot.

    There is a tremendous incentive to produce research with the “correct” results (the results everybody wants to hear).

    No scientific question has ever received this much scrutiny. To borrow a phrase from William Whewell, there is a ‘consilience of evidence’ when it comes to the science of climate change: multiple, independent lines of evidence converging on a single coherent account. These forms of evidence are both observational (temperature records, ice core samples, etc) and theoretical (thermodynamics, atmospheric physics, etc). Together, these lines of evidence provide a conceptual and scientific backing to the theory of climate change caused by human greenhouse gas emissions that is simply absent for alternative theories, such as that there is no change or that the change is caused by something different.

    While there are certainly things we have yet to learn about climate, we have good reason to be confident about the key facts. Greenhouse gas concentrations are rising, that causes warming, humans are the cause of the increase, and there is reason to be concerned about the effects.

    For climate change scientists, their field would lose tremendous amounts of funding if they ever came to the conclusion that climate change wasn’t anything to worry about.

    Surely, there are far more monetary conflicts of interest on the climate change denial side. Coal, oil, and gas are huge industries – and there are plenty of people who are willing to do whatever it takes to defend them. There is a long history of companies like Shell and Exxon funding climate change deniers, including any scientists willing to very publicly question the strength of the science.

    James Hoggan has a good book discussing this in the Canadian context. The Luntz memo is also worth a look.

    How sure are you that anthropogenic climate change is really the doomsday scenario we are led to believe?

    Sure enough for it to be worth taking action. The paleoclimatic record shows that big swings are possible, and there is good reason to be concerned about what one would do to human civilization. On a risk-management basis, it seems preferable to take action to reduce the risk, rather than gamble on the problem being less serious than we thought.

    One thing to consider is that we will eventually need to move away from fossil fuels anyhow. What climate change mitigation requires is doing so sooner than we otherwise would. Making the change also involves many secondary benefits: better air quality, less dependence on volatile states, etc.

    You may find my one page climate primer and page for waverers useful. I also have a page that tracks arguments I’ve had with climate change deniers.

  11. This is not something I have ever heard.

    From http://en.wikipedia.org/wiki/Water_vapor_feedback#Role_of_water_vapor (and yes, I know I should be skeptical of such a source)

    This article points out two things: (1) increased cloud cover is both traps heat and blocks heat. It does not say anything about which effect is more important, and how exactly they scale with temperature. (2) As the temperature of Earth increases, the blackbody spectrum shifts, meaning more ultraviolet and less infrared radiation is emitted. Since the greenhouse effect relies on incoming ultraviolet light being emitted as infrared light and then trapped, this is another mitigating factor.

    Honestly, I’m stretching, and slightly playing devil’s advocate. Also, as I said, I would never advocate against taking precautionary measures, and fully believe our dependence on fossil fuels is bad for a number of other, independent reasons on top of climate effects. That said, while I do not doubt the empirical evidence that human activity has had an effect, arguably already a detrimental one, I am sympathetic to the view that our projections are based on a science which can barely predict the weather 24 hours in advance, and hence worth taking with a grain of salt.

    Overall my main worry is that the “unless we do something about it, we will all die” rhetoric has become fairly commonplace and acceptable, and it weakens the argument.

  12. Also, thanks for the links. I have had a brief look and will try to read in more detail later.

  13. Also, that dig at weather forecasting was pretty uncool. I realize that predicting micro-effects and macro-effects are different, and the latter is much easier. I should really just say that the climate is a complex dynamical system which is hard to predict.

  14. I also have a post on why climate change could be catastrophic. There is no guarantee that human emissions could kick off something as bad as the Permian-Triassic extinction event (which involved a spike in GHG concentrations, and which killed more than 90% of life on Earth), but it cannot be said to be entirely impossible either.

    Bear in mind, also, that a temperature increase of more than 5°C by 2100 is what we would expect from business-as-usual, not some wild scenario. A planet that is 5°C warmer is a transformed world, and one that would cause enormous problems for humanity.

  15. This article points out two things: (1) increased cloud cover is both traps heat and blocks heat. It does not say anything about which effect is more important, and how exactly they scale with temperature. (2) As the temperature of Earth increases, the blackbody spectrum shifts, meaning more ultraviolet and less infrared radiation is emitted. Since the greenhouse effect relies on incoming ultraviolet light being emitted as infrared light and then trapped, this is another mitigating factor.

    Ah, so the negative feedback in question is clouds, not transparent water vapour. Clouds are complex, though their net effect on radiative forcing has been estimated in each IPCC report. While it may be a logical possibility that the reflectiveness of increased clouds would offset the warming from decreased radiation into space, this certainly isn’t considered likely by the IPCC, G8 national science academies, and others. Also, even if the increased cloudiness did have a cooling effect at the perfect level to negate the warming, rising CO2 concentrations would still make the planet’s oceans more acidic, with potentially severe consequences. Also, even if the clouds zeroed out the temperature change, there could still be significant changes in regional climates, precipitation patterns, etc. For instance, the loss of glaciers and summer snowpack in the Himilayas may eventually have a severe impact on agricultural production in Asia.

    One other thing: it’s not just ultraviolet light that warms the planet, causing it to emit infrared light. The entire spectrum of light emitted by the sun has this effect. Wien’s Displacement Law holds that every object in the universe emits electromagnetic radiation, and that the most common frequency exists in relation to that object’s temperature in degrees Kelvin. To go from one to the other, divide 2898 by the temperature in degrees Kelvin. The quotient is the peak wavelength, expressed in microns. Human body temperature is about 310 degrees Kelvin, so our peak electromagnetic wavelength is about 9.35 microns long – in the infrared portion of the electromagnetic (EM) spectrum. Since we are pretty similar in temperature to the surface of the Earth, the wavelengths radiated by the planet are in a nearby portion of the spectrum.

    It is is ability of greenhouse gasses to absorb this infrared energy that lets them prevent energy from returning to space. They are transparent to the dominant wavelengths emitted by the sun, but opaque to those radiating from the Earth. Increasing their concentrations in the atmosphere (through fossil fuel burning, deforestation, etc), causes more of the energy that comes to the Earth from the sun to remain in the atmosphere. As a result of the extra energy, the temperature rises. Incidentally, this is also why people sometimes mention using ground-based mirrors to fight climate change. They reflect light at the same peak wavelength as that of the sun (which passes relatively unimpeded through the atmosphere). By re-radiating at that visible wavelength, rather than the infrared one favoured by greenhouse gasses, the energy can be made to escape again. Of course, it would take a massive number of mirrors to balance out the effect of increased greenhouse gas concentrations on the EM emissions from all non-mirrored areas.

    I am sympathetic to the view that our projections are based on a science which can barely predict the weather 24 hours in advance, and hence worth taking with a grain of salt.

    We would all prefer certainty, but it just isn’t possible with a system this complex and only one chance to observe it.

    What we need to do is make a series of choices in the face of risk and uncertainty. What we know about the climate system seems more than alarming enough to warrant an accelerated transition to a zero-carbon global economy.

  16. The head of the Met Office recently published an article in The Guardian stressing the urgent need to cut greenhouse gas emissions:

    Even if emissions start to decrease in the next two years and reach a rapid and sustained rate of decline of 3% per year, temperatures are likely to rise to 1.7C above pre-industrial levels by 2050 and to around 2C by 2100. This is because carbon dioxide already in the atmosphere will be around for many years to come and the climate takes some time to respond to these changes. Only an early and rapid decline in emissions gets anywhere close to the target of 50% reduction in emissions by 2050 put forward by the G8.
    Contrast that with a world where no action is taken to curb global warming. Then, temperatures could rise as high as 7C above pre-industrial values by the end of the century. This would lead to significant risks of severe and irreversible impacts.

    Also: “In the somewhat understated language typical of scientists, the head of the Met Office has said that warming of this scale would “lead to significant risks of severe and irreversible impacts.” That isn’t a worst-case scenario, but rather their best guess about where we will end up unless we change course. It should also be noted that there are positive feedbacks not incorporated into models such as that of the Hadley Centre: notable among them methane from permafrost. With such feedbacks factored in, a significantly worse business-as-usual warming profile is possible.”

  17. First of all, it’s unfortunate this is being derailed, as it’s a good topic to discuss.

    I have my own theories, developed from extended discussions with people from older generations. The biggest reason people deny climate change is because it demands a massive shift in how we understand the world – instead of merely living in it, we are an increasingly large part of it, and cannot dwell within it without changing it significantly. The demographic shifts which have caused that are relatively new: the global population has more than doubled since our parents were born, and global meat production has increased 400% since 1970. This kind of exponential growth is incomprehensible to most, but it is the world we live in. Instead of trying to change the world, we now struggle to merely preserve it – and that task is getting harder every day.

    I think this quote sheds a little light on the comprehension gap:

    “I used to work for a boss who liked to tell everyone:
    ‘If you always do what you’ve always done, you’ll always get what you’ve always got.’
    I could never make him understand that this was no longer a threat, but a promise, and moreover, one which could not be kept.”

  18. As long as things are comfortable enough, most people will choose to ignore climate change predictions rather than alter not just their pattern of consumption but their ideology as well – the notion that government is the problem, not the solution.
    I doubt very much that it is based on skepticism of the science, because you don’t even need models or a working knowledge of physics to see that glaciers and ice caps are melting, and seasons are changing.
    I rather think those of us who are alarmed would do well to emphasize the other reasons to reduce emissions, rather than the incremental increase in average temperatures primarily (so far) due to rising CO2.
    Chief among them would be the easily demonstrable acidification of the ocean, which doesn’t require a degree in chemistry to understand, and is explained in a very accessible narration of this film: http://witsendnj.blogspot.com/2009/10/you-cant-fish-and-not-have-hope.html One of the most important points is that this is going to result in a collapse of the food chain and, unfortunately, much of the oxygen we humans need to breathe comes from life in the sea.
    The “other” greenhouse gases – nitrous oxides and sulfur dioxides – that are the precursors to ozone and acid rain, are well known to cause cancer in humans, and reduce crop yields. These connections should be emphasized in the debate, because practically everybody now knows somebody with cancer, and just about everybody likes to have food to eat.

  19. It really does not matter what denialists believe. The warming will be proceeding as modeled. Delusional fantasies not withstanding.

    In legal procedures in Kivalina v. Exxon – the US Corps of Engineers confirms that AGW is caused by carbon emissions from carbon fuel combustion – there are other liability cases moving through the courts now.

    Denialists have succeeded in inflicting delay, but time is running out on that tactic. It is kind of us to be polite to denialists, but they are increasingly inconsequential.

  20. What denialists believe matters a lot, because they continue to be politically influential. We’re not talking about some band of crazies off in an armed compound in the woods here – the ranks of hardcore deniers include plenty of influential politicians, and lots of the corporations and fundraisers who they are beholden to.

    If denialists were so inconsequential, would the weak cap-and-trade system proposed by the Obama administration be so bogged down? Would Copenhagen have failed so decisively? Would annual emissions still be rising almost everywhere?

  21. Indeed. Climate change denial remains a huge problem. Otherwise, how can you explain how many people think Climategate is a good reason for avoiding action on mitigation?

  22. And now the denialsphere is frothing at the mouth about the Himalayan glaciers! I expect this to be the new climategate, and they’ll be playing it up while California is deluged with violent storms.

  23. “It really does not matter what denialists believe. ”

    Maybe. It matters what they believe only if belief has something to do with action.

  24. yes, yes, yes, It is the difference between belief and action.

    My doctor can believe in astrology, but if he damn well better not use it in medicine.

    Denialist beliefs are harmless until they are translated into sabotage. Then it is almost like they really don’t believe it, because it appears they are just using that belief to wield PR power. It becomes a phony adopted belief.

  25. The dogma that action is the result of intention or desire plus belief is not serving us well here.

  26. This article sheds the most light for me on the problem of denial from a human psychology perspective:

    http://www.greatchange.org/ov-catton,denial.html

    When the problems are really really big, its easier to be in denial than a more manageable small problem that we can readily obsess over. Like “anosognosia” – what some stroke victims suffer from.

  27. Tristan: “Stopping climate change by reducing your personal carbon footprint is like trying to expand the welfare state by voluntarily paying higher taxes.”

    Well put, BTW.

  28. Scientists ‘losing climate fight’

    A leading Australian climate change scientist says experts are losing the fight against sceptics, who are distorting the science of global warming.

    Professor Pitman was a lead author on the IPCC’s 2001 and 2007 reports. He is also the co-director of the Climate Change Research Centre at the University of New South Wales.

    Professor Pitman says sceptics have used the IPCC’s error to skew the climate change debate.

    “Climate scientists are losing the fight with the sceptics,” he said.

    “The sceptics are so well funded, so well organised. “They have nothing else to do. They don’t have day jobs so they can put all their efforts into misinforming and miscommunicating climate science to the general public, whereas the climate scientists have day jobs and [managing publicity] actually isn’t one of them.

    “All of the efforts you do in an IPCC report is done out of hours, voluntarily, for no funding and no pay, whereas the sceptics are being funded to put out full-scale misinformation campaigns and are doing a damn good job, I think.

    “They are doing a superb job at misinforming and miscommunicating the general public, state and federal governments.”

    And he says if scientists lose the climate change debate, it would be “potentially catastrophic”.

    “If this was academic debate over some trivial issue [it wouldn’t matter],” he said.

    “But this isn’t. This is absolutely a fundamental problem for the Earth that we desperately needed full-scale international action on a decade ago.

    “We are now 10 years too late to stop some of the major impacts that we will see and have seen as a consequence of global warming. It is not a future problem, it is a problem here today, around us.”

    Professor Pitman has accused sceptics of failing to base their arguments on the facts.

    “Most of the climate sceptics, particularly those that are wandering around publicly at the moment, don’t base their arguments on science,” he said.

    “They have probably never read the Intergovernmental Panel on Climate Change report; they aren’t writing papers in peer-reviewed literature.

    They don’t update their arguments when their arguments are shown to be false, so they’ll have no problem at all using this ammunition inappropriately and out of context to further their aims in exactly the same way as people did when they were trying to disprove the relationship between smoking and human health.”

  29. The battle we are losing is only a political and informational skirmish. In the great war, the science and the laws of thermodynamics remain solid – and will be victorious.

    Faced with such organized denialism – one can move into feeling a type of species self contempt. How can we persist in our stupidity? Unfortunately that very attitude breeds either violent resolutions or passively destructive ends. Bang or whimper.

    In order to survive we will need to evolve into beings who act wisely, otherwise after just a few more generations, we face extinction. Unlike any other known species, humans must now use higher thinking and technology in order to re-define ourselves into a new species .

    I can see the potential, but I wish I could see more promising beginnings.

  30. The Six Americas of Climate Change
    Posted by Clark Williams-Derry

    My favorite is this: folks who are convinced that global warming is a hoax — the “Dismissives” — admit they haven’t thought all that much about the issue (see Figure 6 on page 14 of the pdf) yet rank themselves as extremely knowledgeable and well informed (see Figure 7).

    That should tell us something: for many climate skeptics, facts don’t matter much. They’ve only given the subject a bit of thought, but are still convinced that they know the answers. I don’t mean to be snarky, but to me this suggests that some “Dismissives” may suffer from some version of the Dunning-Kruger effect — the idea that people are very poor judges of their own incompetence. That probably makes many “Dismissives” unreachable: when facts confront their biases, the facts bounce off and the biases stand firm. (I’m sure that’s true of us all, to some degree or another.)

  31. In his latest book, James Hansen describes the reasoning of prominent climate change denier (and MIT meteorology professor) Richard Lindzen:

    Lindzen’s perspective on climate sensitivity, as he told Richard Kerr, stems from an idea of a theological or philosophical perspective that he doggedly adheres to. Lindzen is convinced that nature will find ways to cool itself, that negative feedbacks will diminish the effect of climate forcings. p.55 (hardcover)

    This reminds me of something I wrote in a forthcoming book review: “Nor does [Mike Hulme] consider that religious belief might diminish concern about climate change, since an orderly universe with a creator seems less likely to allow humanity to capriciously extinguish itself than one governed only by chance and physical laws.”

    If someone starts out with the assumption that the Earth will remain a hospitable and even benevolent environment for human beings, one naturally has to discount the possibility that climate change would alter that.

  32. In our earlier argument, I think climate change denier Dan Pangburn made a similar assumption, and he never adequately responded when I called him out on it:

    “You keep asserting that there is some phenomenon counteracting this, producing your infinitely repeated situation of “no significant NET positive feedback [do you mean radiative forcing?] from average global temperature.” What specific mechanism counteracts the infrared absorbing effect of greenhouse gasses? If such an effect exists, why has it automatically been getting stronger as concentrations rise? Also, what proof is there that even if there were such an effect, it would protect us from any amount of increased GHG concentrations. For instance, continued business-as-usual emissions could push concentrations to over 1000 ppm of CO2 equivalent by 2100, compared to 280 ppm before the Industrial Revolution and about 383 ppm now. Even if there were negative feedback effects that significantly reduced the total forcing resulting from increased GHG concentrations (that is, lowered climatic sensitivity), it is possible that they would break down when presented with such a significant change.”

  33. That is a beautiful example of begging the question:

    Lindzen: “Climate change won’t be dangerous for people.”

    Hansen: “But what about the warming effect from all the greenhouse gases we are emitting? Isn’t that likely to cause droughts, heatwaves, rising sea levels, and so forth?”

    Lindzen: “No, because there is a negative feedback effect that counteracts the positive ones that worry you.”

    Hansen: “Really? What sort of effect? How can you be sure it exists?”

    Lindzen: “We know it exists because climate change won’t be dangerous for people.”

  34. Writing about an image of the Earth, taken from deep space, Carl Sagan may have expressed the rebuttal to Lindzen’s view most elegantly:

    Our posturings, our imagined self-importance, the delusion that we have some privileged position in the universe, are challenged by this point of pale light.

    Our planet is a lonely speck in the great enveloping cosmic dark. In our obscurity — in all this vastness — there is no hint that help will come from elsewhere to save us from ourselves. It is up to us.

    Hopefully, humanity can learn that lesson, before we emit enough greenhouse gases to commit future generations to a bleak existence, or even no existence at all.

  35. I think Zizek’s account of climate change non-attention (which includes climate denial, but climate denial alone can probably not explain our inaction), which is a psycho-analytic one, is the most convincing.

  36. If you prefer text, I will try to find you a paper where he argues for this. However, the last time I tried to send you any text by a post-marxist it did not go well.

  37. People jumped on ClimateGate because it provides such an easy out.

    “For a moment there, it looked like we were actually going to have to change our lifestyles. Thankfully, we can use the excuse provided by these emails (and other minor errors in IPCC reports) to say that we need to start the science over.”

    It’s like a smoker rejoicing in any news story that hints that smoking and cancer may not be linked, or that smoking is somehow beneficial. It was easy to ‘sell’ the ClimateGate story because everyone was already eager to buy it.

  38. It also provides a handy excuse for later. In twenty or thirty years, when the Greenland and West Antarctic ice sheets are disintegrating, people can excuse themselves for not taking action back in 2010 because the science was supposedly so unclear.

  39. RPauli, with that comment you have provided conclusive proof of one of two possibilities:

    1. You are pure genius at finding obscure and diverting links; or

    2. You have to much time on your hands. Have you finished regrouting the bath?

    Or perhaps both…

  40. “People who study irrational beliefs have a variety of ways of explaining why we cling to them. In rational choice theory, what appear to be crazy choices are actually rational, in that they maximize an individual’s benefit—or at least make him or her feel good.

    Another explanation for the refusal to face facts is what cognitive scientists call confirmation bias. Years ago, when writing an article for the Washington Post Magazine about the Tailwind affair, a screwy piece of journalism about a nonexistent attack on American POWs with sarin gas, I concluded that the story’s CNN producers had become wedded to the thesis after interviewing a few unreliable sources. After that, they unconsciously discounted any facts that interfered with their juicy story. They weren’t lying—except, perhaps, to themselves. They had brain blindness—confirmation bias.”

  41. Climate Change Denial

    This blog explores the topic of the psychology of climate change denial – with observations and anecdotes about our weird and disturbed response to the problem. It seeks to answer a question that has puzzled me for years: why, when the evidence is so strong, and so many agree that this is our greatest problem, are we doing so little about climate change?

  42. Denial that climate change is human caused follows the liberal/conservative axis with most people who deny climate change
    identifying themselves as conservative. It seems that people tend to
    adhere so strongly to the ideology of their position that they deny the
    validity of all arguments on the other side. Since human caused climate change is rightly or wrongly associated with liberalism, conservatives are compelled to disagree. Climate change requires large changes to the status quo to which conservatives are constitutionally opposed.

  43. “There is no such thing at this date of the world’s history in America as an independent press. You know it, and I know it. There is not one of you who dares to write his honest opinion, and if you did, you know beforehand it would never appear in print. I am paid weekly for keeping my honest opinion out of the paper. Others of you are paid similar salaries for similar things. and any of you who would be so foolish as to write honest opinions would be out on the streets looking for another job. If I allow my honest opinions to appear in one issue of my paper, before 24 hours, my occupation would be gone. The business of the journalist is to destroy the truth, to lie outright, to pervert, to vilify, to fawn at the feet of Mammon and to sell his country and his race for his daily bread. You know it, and I know it, and what folly is this toasting an independent press? We are the tools and the vassals of rich men behind the scenes. We are the jumping jacks. They pull the strings, and we dance. Our talents, our possibilities and our lives are all the property of other men. We are intellectual prostitutes.”

    — John Swinden, 1953, then head of the New York Times, when asked to toast an independent press in a gathering at the National Press Club

  44. haha I love that quote, Just Some News!!! I may have to steal it and post it elsewhere – it deserves wide disbursement!

    Q: “why, when the evidence is so strong, and so many agree that this is our greatest problem, are we doing so little about climate change?”

    A: because, everybody is quite rightfully and fundamentally terrified. The globe is now one gigantic Easter Island, and cannibalism and mass suicide are in the offing.

  45. Linking to relevant news

    One thing that I try to do on this site is accompany posts on all topics with links to related materials: both in terms of what I have written and what is out on the wider internet. One way I do this is by leaving comments that link to and quote from relevant news stories and websites. By convention, these comments are attributed to ‘.’ since it doesn’t take long to write and cannot easily be confused with a real person.

  46. Gail, Great comment.

    A. 1. Actively denying such a situation, is a military-like tactic where the opportunists gain the advantage of surprise. The denialists are harvesting and plundering (like banks hoarding money – refusing to loan it out) The carbon fuel industry and ALL associated industries can continue at full speed… auto industry, airline, most all advertising – all must buy in to the lie. Their reward is tremendous profit.

    I would forgive denialism due to the psychological difficulty of looking at a cruel future. But that is essentially passive denial. The strident denialism really belies and understanding of the truth.

  47. Belief In Climate Change Hinges On Worldview
    by Christopher Joyce

    Over the past few months, polls show that fewer Americans say they believe humans are making the planet dangerously warmer, despite a raft of scientific reports that say otherwise.

    This puzzles many climate scientists — but not some social scientists, whose research suggests that facts may not be as important as one’s beliefs.

    Take, for example, a recent debate about climate change on West Virginia public radio.

    “It’s a hoax,” said coal company CEO Don Blankenship, “because clearly anyone that says that they know what the temperature of the Earth is going to be in 2020 or 2030 needs to be put in an asylum because they don’t.”

    On the other side of the debate was environmentalist Robert Kennedy, Jr.

    “Ninety-eight percent of the research climatologists in the world say that global warming is real, that its impacts are going to be catastrophic,” he argued. “There are 2 percent who disagree with that. I have a choice of believing the 98 percent or the 2 percent.”

  48. The Unpersuadables
    Posted March 8, 2010

    In fighting for science, we subscribe to a comforting illusion: that people can be swayed by the facts.

    By George Monbiot. Published in the Guardian 9th March 2010.

    There is one question that no one who denies manmade climate change wants to answer: what would it take to persuade you? In most cases the answer seems to be nothing. No level of evidence can shake the growing belief that climate science is a giant conspiracy codded up by boffins and governments to tax and control us. The new study by the Met Office, which paints an even grimmer picture than the Intergovernmental Panel on Climate Change, will do nothing to change this view.

    The attack on climate scientists is now widening to an all-out war on science. Writing recently for the Telegraph, the columnist Gerald Warner dismissed scientists as “white-coated prima donnas and narcissists … pointy-heads in lab coats [who] have reassumed the role of mad cranks … The public is no longer in awe of scientists. Like squabbling evangelical churches in the 19th century, they can form as many schismatic sects as they like, nobody is listening to them any more.”

    Views like this can be explained partly as the revenge of the humanities students. There is scarcely an editor or executive in any major media company – and precious few journalists – with a science degree, yet everyone knows that the anoraks are taking over the world. But the problem is compounded by complexity. Arthur C Clarke remarked that “any sufficiently advanced technology is indistinguishable from magic”. He might have added that any sufficiently advanced expertise is indistinguishable from gobbledegook. Scientific specialisation is now so extreme that even people studying neighbouring subjects within the same discipline can no longer understand each other. The detail of modern science is incomprehensible to almost everyone, which means that we have to take what scientists say on trust. Yet science tells us to trust nothing; to believe only what can be demonstrated. This contradiction is fatal to public confidence.

  49. Too bad that scientists suffer the weaknesses of any human being.

    So the oil and coal industry employ thousands of scientists – geologists mostly — and few want to speak out against their employer. Then academic scientists retreat to narrowly defined specialties and dare not comment outside their fields. So to get a decent overview of AGW, one has to stitch together research reports from numerous specialties. Finally there are uncounted numbers of scientists who see clearly the problem, but see no clear resolution and so refuse to step forward or speak out.

    The world has seen this kind of anti-science drama replayed in the past with deadly squabbles over flat earth, heliocentrism, aether, plate tectonics, and evolution. The only difference is this an existentialist drama of the future of our species.

  50. If you are looking for some causes here are some
    – a post-Watergate, post-Reagan culture with its unquestioned expectation of an officialdom that is assumed to be corrupt,wasteful, manipulative and predatory — where we collectively are assumed to be its victims
    – the growth of brutally tribal blog ghettos where anonymity, viciousness and slagging enforce and re-inforce groupthink. Winston Churchill once said that a lie can travel the globe before the truth gets its pants on. The Internet has made this not just robustly viable, but virulent
    – a soundbite media environment where so called entertainment is king and where ideas that take longer than 30 seconds to digest simply die from lack of oxygen.
    – the discrediting of and disconnection from science in day-to-day life
    – a modern urban life where many spend way more time in front of a screen than in the direct experience of nature.

    I agree that the term “Climate Change Skeptic” conceals more than it reveals. Perhaps something like “Climate Change Counter-revolutionary” is closer to reality. Or, given that much of the back-and-forth (I won’t use the word ‘debate’) is framed in the language of belief rather than the language of science, how about “Climate Change Heretic”?

  51. Brien, I like climate change heretic except maybe the religious connotations aren’t so useful. RPauli suggested climate change cretin which is pretty good too!

    I agree with everything you said however, as one who dwells in the country I have to say that the people in farm country are still perversely oblivious to the significance of the changes in nature. It’s quite bizarre actually.

  52. The Trouble With Climate Science
    More research makes the controversy worse.
    By Daniel Sarewitz
    Updated Wednesday, March 10, 2010, at 5:18 PM ET

    Has anyone noticed that after 20 years and $25 billion in government-sponsored research on climate change, the political controversy over global warming is actually more intractable and bitter today than it has ever been in the past? Of course there are good reasons, such as the failure of the recent U.N.-sponsored Copenhagen climate conference, stubborn partisanship in the U.S. Congress, and recently discovered mistakes and distortions in the supposedly authoritative 2007 report of the Intergovernmental Panel on Climate Change. But these, like the tumors in a cancer patient, are symptoms of a mortal pathology, not its cause.

    A dangerous idea has taken hold in modern politics, and the sooner it is discredited, the better. The idea is that political disagreements can be resolved by science. Its basic logic seems sensible: As good children of the Enlightenment, we should turn to science to establish the facts about problems such as climate change before deciding what policies to implement. Yet the types of things that scientists are good at figuring out don’t have much to do with the types of things that politicians need to decide.

    If this point seems seditiously anti-rational, consider the recent history of environmental politics in the United States. In the late 1960s and early 1970s, Congress enacted an impressive raft of laws addressing air and water quality, endangered species, pesticide use, toxic waste clean-up, and the environmental impacts of government projects. The state of scientific knowledge at that time was primitive relative to today. But the political climate was favorable, and many of the problems— like smog and burning rivers—were obvious for all to see, so the science was more than sufficient to support action. Laws were passed, regulations were promulgated, and environmental protection was advanced. Four decades later, scientific understanding of the environment has improved immeasurably, while political action has become almost impossible. More knowledge has created more uncertainty about what works and what doesn’t, about what’s a problem and what isn’t, and has given more ammunition to competing positions on issues ranging from protection of endangered species to regulation of toxic chemicals in the environment.

    The most wonderful illustration of this mismatch between what science can tell us and what politicians care about is the effort to build a long-term storage site for nuclear waste at Nevada’s Yucca Mountain. It’s probably fair to say that, after 25 years and $13 billion of government-funded research, no area of ground on Earth is more studied than Yucca Mountain, yet all of this science has done absolutely nothing to quell opposition from locals and environmental groups. On the contrary, it provided a continual source of new discoveries and uncertainties that combatants could draw upon to bolster their political and legal cases. For example, varying estimates of the amount of ground water flowing through the rocks at the site were central both to claims that Yucca Mountain was safe and that it should be abandoned.

  53. Elizabeth E. May: Don’t demonize the messenger

    Climate scientists are now in a maelstrom of competing caricatures. In the Post last week, Patrick Keeney (“Trust us, we’re experts,” March 5) described them as schemers, cooking the books, using PR and spin. Those who wish to believe the climate crisis is not the result of human activity herald as heroes scientists who disagree with the consensus view, while others deride them as corrupt.

    The truth is that scientists are a bunch of different kinds of people. They tend toward knowing their own area of work in depth, having a distrust of politics and media, and possessing a healthy degree of skepticism.

    In 1986, when I worked as senior policy advisor to the minister of environment in the Mulroney government, departmental briefings on climate science were alarming. Even then, the fundamentals were well understood. Burning fossil fuels was releasing unprecedented levels of greenhouse gases into the atmosphere. The “greenhouse effect” was suddenly converting what had been a natural and beneficial phenomenon into a menace. The first major global scientific climate conference, held in Toronto in June 1988, concluded, “Humanity is conducting an unintended, uncontrolled, globally pervasive experiment whose ultimate consequences could be second only to global nuclear war.”

    That year, governments developed a reliable body to assess the science — the UN Intergovernmental Panel on Climate Change. Some 2,000 scientists were appointed by governments to review the peer-reviewed work of thousands of others. Reading the layman’s summary section of any IPCC assessment report should be enough to convince anyone that these scientists are not spin doctors. For example, from the 2001 Summary for Policy Makers: “The global average surface temperature has increased by 0.6 + or – 0.2 degrees C since the late 19th century. Footnote: generally temperature trends are rounded to the nearest 0.05 degrees C per unit of time, the periods often being limited by data availability.”

    They are also ethical, hard working and honest. The literature is rife with differing opinions, but these differences are on the edges of the science. No legitimate dispute remains about the fact that we have changed the chemistry of the atmosphere and that the results are already being witnessed in a destabilized climate. Based on Antarctic ice-core data, science that no one has disputed, the atmosphere now contains more than 30% more carbon dioxide than at any time in the last million years. There is no explanation for this other than human activity. And, if our addiction to fossil fuels continues, levels will keep rising with disastrous results.

  54. I put together an article some months ago about people’s response to the issue of climate change which contains some ideas that I think are relevant to the current discussion: http://petrolog.typepad.com/climate_change/2009/11/peoples-response-to-climate-change.html. The article also includes a link to a paper from the World Bank that identifies ways in which the public responds when confronted with news and information about climate change:

    People find it hard to frame a response when exposed to too much conflicting information.
    The man in the street is not moved by the delay and uncertain outcome of the problem.
    Humans strongly discount the future and assign higher priorities to more immediate problems.
    People commonly fail to act on available information if the cost of making a decision is too high.
    People construct and re-construct information to make it less uncomfortable.

  55. “People commonly fail to act on available information if the cost of making a decision is too high.
    People construct and re-construct information to make it less uncomfortable.”

    This is deeply true – it’s part of the existential structure of experience (you are finite and the world is infinite, and you have no choice but to ignore most of it and hope you can cope with the simplifications you can manage). However, not making a decision, hiding from the infinite complexity of the world behind a bush – this might be the nature of evil.

  56. “You might say that some see a jigsaw where others see a house of cards. Jigsaw types have in mind an overall picture and are open to bits being taken out, moved around or abandoned should they not fit. Those who see houses of cards think that if any piece is removed, the whole lot falls down. When it comes to climate, academic scientists are jigsaw types, dissenters from their view house-of-cards-ists.

    The defenders of the consensus tend to stress the general consilience of their efforts—the way that data, theory and modelling back each other up. Doubters see this as a thoroughgoing version of “confirmation bias”, the tendency people have to select the evidence that agrees with their original outlook. But although there is undoubtedly some degree of that (the errors in the IPCC, such as they are, all make the problem look worse, not better) there is still genuine power to the way different arguments and datasets in climate science tend to reinforce each other.

    The doubters tend to focus on specific bits of empirical evidence, not on the whole picture. This is worthwhile—facts do need to be well grounded—but it can make the doubts seem more fundamental than they are. People often assume that data are simple, graspable and trustworthy, whereas theory is complex, recondite and slippery, and so give the former priority. In the case of climate change, as in much of science, the reverse is at least as fair a picture. Data are vexatious; theory is quite straightforward. Constructing a set of data that tells you about the temperature of the Earth over time is much harder than putting together the basic theoretical story of how the temperature should be changing, given what else is known about the universe in general.”

  57. Living in denial: Why sensible people reject the truth

    HEARD the latest? The swine flu pandemic was a hoax: scientists, governments and the World Health Organization cooked it up in a vast conspiracy so that vaccine companies could make money.

    Never mind that the flu fulfilled every scientific condition for a pandemic, that thousands died, or that declaring a pandemic didn’t provide huge scope for profiteering. A group of obscure European politicians concocted this conspiracy theory, and it is now doing the rounds even in educated circles.

    This depressing tale is the latest incarnation of denialism, the systematic rejection of a body of science in favour of make-believe. There’s a lot of it about, attacking evolution, global warming, tobacco research, HIV, vaccines – and now, it seems, flu. But why does it happen? What motivates people to retreat from the real world into denial?

    Here’s a hypothesis: denial is largely a product of the way normal people think. Most denialists are simply ordinary people doing what they believe is right. If this seems discouraging, take heart. There are good reasons for thinking that denialism can be tackled by condemning it a little less and understanding it a little more.

    Whatever they are denying, denial movements have much in common with one another, not least the use of similar tactics (see “How to be a denialist”). All set themselves up as courageous underdogs fighting a corrupt elite engaged in a conspiracy to suppress the truth or foist a malicious lie on ordinary people. This conspiracy is usually claimed to be promoting a sinister agenda: the nanny state, takeover of the world economy, government power over individuals, financial gain, atheism.

  58. “All denialisms appear to be attempts like this to regain a sense of agency over uncaring nature: blaming autism on vaccines rather than an unknown natural cause, insisting that humans were made by divine plan, rejecting the idea that actions we thought were okay, such as smoking and burning coal, have turned out to be dangerous.

    This is not necessarily malicious, or even explicitly anti-science. Indeed, the alternative explanations are usually portrayed as scientific. Nor is it willfully dishonest. It only requires people to think the way most people do: in terms of anecdote, emotion and cognitive short cuts. Denialist explanations may be couched in sciency language, but they rest on anecdotal evidence and the emotional appeal of regaining control.”

  59. “We’d like to believe that most of what we know is accurate and that if presented with facts to prove we’re wrong, we would sheepishly accept the truth and change our views accordingly.

    A new body of research out of the University of Michigan suggests that’s not what happens, that we base our opinions on beliefs and when presented with contradictory facts, we adhere to our original belief even more strongly.

    The phenomenon is called backfire, and it plays an especially important role in how we shape and solidify our beliefs on immigration, the president’s place of birth, welfare and other highly partisan issues.”

  60. Whatever the causes, skepticism about the seriousness of climate change is growing in the United States.

    While the accusations associated with the leaked CRU emails are unfounded, they are almost certainly part of the explanation for this shift.

  61. “It’s the same story everywhere. At a senatorial hustings in New Hampshire last week, all six Republican candidates denied that man-made climate change is taking place. Judging by its recent antics in the Senate and by primary campaigns all over the country, the Republican party appears to be heading towards a unanimous rejection of the science. The ultra-neoliberal Czech president Vaclav Klaus asserts that “global warming is a false myth and every serious person and scientist says so.” The hard-right UK Independence Party may soon be led by Lord Monckton, the craziest man in British politics, who claims that action on climate change is a conspiracy to create a communist world government. The further to the right you travel, the more likely you are to insist that man-made climate change isn’t happening. Denial has nothing to do with science and everything to do with politics.

    Yes, man-made climate change denial is about politics, but it’s more pragmatic than ideological. The politics have been shaped around the demands of industrial lobby groups, which happen, in many cases, to fund those who articulate them. Right-wingers are making monkeys of themselves over climate change not just because their beliefs take precedence over the evidence, but also because their interests take precedence over their beliefs.”

  62. “Meet the Climate Zombies.

    They’re mindless.

    Their stupid is contagious.

    And if they win, humanity loses.

    A couple of weeks ago, the Wonk Room had a story: Every GOP N.H. Senate candidate is a global warming denier. At a candidates’ forum in Portsmouth, N.H., “all said man-made global warming hasn’t been proven.”

    The epidemic next appeared in New Mexico, where all three Republican candidates for Congress, and the GOP candidate for governor, denied the existence of man-made climate change. The candidates for Congress gave waffling-but-cool answers on a questionnaire, but subsequent digging revealed that all flatly deny the science.

    Intrigued, I began to poke around other states. Virtually all Republicans criticize what they call “cap-and-tax” as too expensive, but how many actually deny the reality of climate change science? How many have been infected by Teh Stoopid?

    A lot. A real lot. Be afraid.

    n conclusion: We sampled four states with a total of 22 representatives, three gubernatorial candidates (excluding Wisconsin), and three senators up for reelection. Four representatives (Okla.-02, Okla.-03, Okla.-04,and Wis.-06) seem to accept the reality of climate science, if not the solution; two (Ariz.-01, Ariz.-05) have been silent to date; three (Wis.-03, Wis.-07, and Wis.-08) haven’t been selected yet; and 13 express skepticism/hostility. Of the three candidates for governor, one is openly hostile and two are silent. Of the three candidates for Senate, two are openly hostile and the third is John McCain.

    Climate zombies are now the Republican party norm.”

  63. “But harping and nagging and saying “See how bad fossil fuels are” doesn’t really get us anywhere.

    Here’s why: People already know that our dependence on dirty energy is a problem. They get it. There’s even an appetite for a national clean-energy strategy/program — it’s just not overwhelming enough to overcome the undemocratic dynamics of the Senate.

    So why aren’t we sufficiently riled up to change the situation? One of the more persuasive theories is status quo bias. Psychologists find that when people are anxious (and nothing breeds anxiety like a recession), they cling to what they know, even if it’s clearly problematic. Our fossil-fuel economy may be gasping, sputtering, and occasionally blowing up homes, but at least it’s the devil we know, not some mysterious cleantech network of solar panels and smart meters and compact neighborhoods.

    People don’t need hectoring told-you-so’s. They need to see a vision of a sustainable future that actually looks appealing.

  64. “Hanging over everything is the growing recognition that the United States isn’t going to play. Not this year, perhaps not in any year. If Congress couldn’t pass a climate bill so feeble that it consisted of little but loopholes while Barack Obama was president and the Democrats had a majority in both houses, where does hope lie for action in other circumstances? Last Tuesday the Guardian reported that of 48 Republican contenders for the Senate elections in November only one accepted that manmade climate change is taking place. Who was he? Mike Castle of Delaware. The following day he was defeated by the Tea Party candidate Christine O’Donnell, producing a full house of science deniers. The Enlightenment? Fun while it lasted.

  65. “Overall, we found that 63 percent of Americans believe that global warming is happening, but many do not understand why. In this assessment, only 8 percent of Americans have knowledge equivalent to an A or B, 40 percent would receive a C or D, and 52 percent would get an F. The study also found important gaps in knowledge and common misconceptions about climate change and the earth system. These misconceptions lead some people to doubt that global warming is happening or that human activities are a major contributor, to misunderstand the causes and therefore the solutions, and to be unaware of the risks. Thus many Americans lack some of the knowledge needed for informed decision-making in a democratic society. For example, only:

    * 57% know that the greenhouse effect refers to gases in the atmosphere that trap heat;
    * 50% of Americans understand that global warming is caused mostly by human activities;
    * 45% understand that carbon dioxide traps heat from the Earth’s surface;
    * 25% have ever heard of coral bleaching or ocean acidification.

    Meanwhile, large majorities incorrectly thinkthat the hole in the ozone layer and aerosol spray cans contribute to global warming, leading many to incorrectly conclude that banning aerosol spray cans or stopping rockets from punching holes in the ozone layer are viable solutions. “

  66. Some encouraging findings from that study:

    Americans also recognize their own limited understanding. Only 1 in 10 say that they are “very well informed” about climate change, and 75 percent say they would like to know more about the issue. Likewise, 75 percent say that schools should teach our children about climate change and 68 percent would welcome a national program to teach Americans more about the issue.

  67. Insofar as lack of public engagement is the problem, the cause is not misinformation, it’s the lack of affective information — information that is meaningful, that speaks to core fears and aspirations. The main problem is apathy. People just don’t care much. Green journos and pundits tend to wildly overestimate the significance of accurate knowledge and wildly underestimate the significance of emotional resonance.

    Those trying to spread the word on climate change have the advantage in numbers. The majority of Americans accept that climate change is happening and almost three-quarters get a passing grade — C or above — on Yale’s scale of knowledge. Where the denialists have the overwhelming advantage is in intensity. As rejection of climate science and climate solutions has become an ideological litmus test on the right, millions of Republicans have come to believe that climate science is not just incorrect but a hoax meant to further U.N. world government. They are pissed.

    Very few of those who correctly believe that climate change is happening are pissed about it. More like “concerned,” the way people are concerned about homelessness or poverty in Africa, like, y’know, somebody (else) should really do something about that. Few write letters to legislators or hassle them about it in town halls. Almost no one will change their vote over it. No legislator stands to be primaried or driven from office over it.

  68. Why don’t Americans believe in global warming?

    I’ve been wanting to take a step back and think about why America is a laggard in the fight against climate change. I would posit a handful of explanations:

    Psychological: The consequences of climate change are too awful to contemplate. Therefore, we’re denying the issue, as we used to deny monsters in the room by hiding under the blanket. If you don’t look at it, it can’t look at you.

    Economic: The costs of a large-scale effort to fight global warming are too steep to bear. Therefore, we’re trying to ignore the issue, or pretending it doesn’t exist, or we believe that the economy (including development) is more important.

    Political: The fact that Democrats are always hammering on about climate change and Republicans aren’t suggests that this is a political issue, not a scientific one. This creates a feedback loop: if climate change were real, why is it so polarising? Because it’s so polarising, it must be slightly suspicious.

    Epistemological: Why should we believe in climate change? Where’s the evidence? All we know is what scientists say, and scientists are sometimes wrong. And don’t even get me started on Al Gore.

    Metaphysical: God isn’t going to let millions of people die in an epic drought.

    I suspect the metaphysical denial is quite rare—but given the comparative religiosity of American culture and the stereotypes thereof, it gets a lot of air time. It is also the least valid of the reasons for denial (partly because in the given system, God obviously does let people die). The other explanations are more common. In the Rasmussen poll, for example, a plurality of respondents said that “there is a conflict between environmental protection and economic growth.”

  69. The truth is that the scientific community has reached a consensus on climate change. The buildup of heat-trapping emissions from burning fossil fuels and clearing forests is changing the climate, posing significant risks to our well-being. Reducing emissions and preparing for unavoidable changes would greatly reduce those risks. That is the conclusion of the U.S. National Academy of Sciences, the world’s leading scientific societies, and the overwhelming majority of practicing climate scientists.

    But many people don’t accept the facts. Social scientists – psychologists, sociologists, and the like – can help us understand why. When they examine climate change, they see what Yale and George Mason University researchers identify as “Six Americas.” They range from “Dismissives,” who are hostile to the science, to the “Alarmed,” who worry that we are running out of time to reduce emissions.

    How can a divided America come together and address climate change? The institutions we work for recently convened social scientists, climate scientists, business leaders, politicians, and faith leaders to help us answer that question.

    According to social scientists, when people hear scientific evidence about societal risks – whether they concern climate change, disposal of nuclear waste, or vaccines – they actively filter it. They accept evidence they find consistent with deeply held cultural values and reject evidence they feel challenges those values.

    A powerful way to break through is for respected leaders to speak out and bring shared values to bear on climate risks and choices. Many people who spoke at the conference are doing just that and helping to build a respectful, fact-based dialogue about climate change.

    http://www.philly.com/philly/opinion/inquirer/20120227_On_climate_change__society_trails_science.html

  70. Climate Change Acceptance Sinks During Economic Slumps

    A few years back, the US public’s acceptance of conclusions reached by climate scientists took a dramatic drop. It’s only now beginning to recover. Not a lot has changed about the science during that time, raising questions about what’s driving the ups and downs in the polls. Studies have found correlations with the weather and a role for political leaders in driving these changes, but a new study suggests some of that is misplaced. Instead, its authors come to a conclusion we’ve heard before: it’s the economy, stupid.

    The authors use polling data from a variety of sources, which creates a bit of a challenge. Not all polls ask questions that address the same things. For example, one of the studies we linked above asked about the public’s acceptance of a basic fact: has our planet been getting warmer over the past few decades? In contrast, one of the polls used here assessed feelings about climate change by asking its participants whether they felt the media “exaggerate the seriousness of global warming.”

    Still, there are ways to convert these specific sentiments into a generalized sense about the seriousness of climate change. Plus, the variety of polls provide some distinct advantages. For example, this survey provides a valuable outgroup to the US population, in that a number of surveys cover all the nations of the European Union. In addition, several of the polls (those performed by the Pew) include ZIP code information, allowing the authors to compare polling trends with record high and low temperatures in the nearby area.

  71. “There’s really only about 25 of us doing this,” Steve Milloy says, shortly after sitting down at Morton’s, a Washington, D.C., steakhouse favored by lawyers and lobbyists. “A core group of skeptics. It’s a ragtag bunch, very Continental Army.” Milloy, a Fox News commentator and former tobacco-industry advocate, runs a website called JunkScience.com that is an outlet for attacks on those he calls “global-warming alarmists.” Many of those who question mainstream climate science resent being called deniers; they say it unfairly equates them with Holocaust deniers. They prefer doubters, skeptics or realists. “Me, I just stick with denier,” Milloy says. “I’m happy to be a denier.”

    Milloy is dressed in a striped pink button-down shirt and khaki pants, classic Potomac prep. He moved into climate denial in the 1990s as funding from the tobacco lobby began to dry up. At the time, conservative and libertarian think tanks were just starting to take aim at climate science. Milloy, who has received funding from entities controlled by oil billionaires Charles and David Koch, helps them get their message to the masses.

    Milloy and other aggressive deniers practice a form of asymmetric warfare that is decentralized and largely immune to reasoned response. They launch what Aaron Huertas, a press secretary at the Union of Concerned Scientists, calls “information missiles,” anti-climate-change memes that get passed around on listservs, amplified in the blogosphere, and picked up by radio talk-show hosts or politicians. “Even if they don’t have much money, they are operating in a structure that allows them to punch above their weight,” Huertas says.

    Scientists who speak up quickly become targets. Both Milloy and his counterpart Marc Morano, who runs the site ClimateDepot.com and once declared that climate scientists “deserve to be publicly flogged,” occasionally publish the e-mail addresses of climate researchers, a stunt that can result in scientists receiving a flood of vitriolic messages. A few weeks before our meeting, Milloy had offered a $500 bounty for a video of anyone who would heckle Mann with “an alarmism-debunking” question during the California leg of his book tour. The hecklers never materialized but, as with the white powder in Mann’s letter (which the FBI determined to be cornstarch), the threat made an impact.

    Mann calls Milloy “a valueless, all-purpose denier for notorious industries who need a hired gun.” But Milloy, like others in the movement, says that he’s fighting an existential war with forces that would, without his intervention, steal the American way of life. “This whole green thing, the whole environmental scare industry, is really just an ingenious plan to exert government control over everything we do,” Milloy says. “I have yet to see an environmental scare that is remotely true when it comes to human health. Secondhand smoke, air quality, ozone depletion, pesticides, superfund sites—you name it.”

    In 1998, following the negotiation of the Kyoto Protocol on global warming, the American Petroleum Institute convened a task force to spend more than $5.9 million to discredit climate science and quash growing public support of curbing emissions. The group borrowed many of the methods and people, including Milloy, that had been used to mislead Congress and the public about the connection between smoking and cancer and heart disease. In a leaked memo titled the “Global Climate Science Communications Plan,” the task force laid out a strategy to “build a case against precipitous action on climate change based on the scientific uncertainty.” The memo details a plan to recruit, train and pay willing scientists to sow doubt about climate science among the media and the public. “Victory will be achieved,” the memo states, when “recognition of uncertainties becomes part of the ‘conventional wisdom’ ” and when “those promoting the Kyoto treaty on the basis of the extant science appear to be out of touch with reality.”

    http://www.popsci.com/science/article/2012-06/battle-over-climate-change?single-page-view=true

  72. Your opinion on climate change might not be as common as you think

    Z. Leviston,
    I. Walker
    & S. Morwinski

    Nature Climate Change (2012) doi:10.1038/nclimate1743

    Political and media debate on the existence and causes of climate change has become increasingly factious in several western countries, often resting on claims and counter-claims about what most citizens really think. There are several well-established phenomena in psychology about how people perceive the prevalence of opinions, including the false consensus effect1 (a tendency to overestimate how common one’s ‘own’ opinion is) and pluralistic ignorance2 (where most people privately reject an opinion, but assume incorrectly that most others accept it). We investigated these biases in people’s opinions about the existence and causes of climate change. In two surveys conducted 12 months apart in Australia (n = 5,036; n = 5,030), respondents were asked their own opinion about the nature of climate change, and then asked to estimate levels of opinion among the general population. We demonstrate that opinions about climate change are subject to strong false consensus effects, that people grossly overestimate the numbers of people who reject the existence of climate change in the broader community, and that people with high false consensus bias are less likely to change their opinions.

  73. The Economist claims to engage in a “severe contest” with “an unworthy, timid ignorance obstructing our progress”. Mr Hirschman was an eloquent ally. In “The Rhetoric of Reaction” he wrote that purveyors of “timid ignorance” rely on three types of argument: jeopardy (reforms will cost a lot and endanger previous gains); perversity (reforms will harm the people they are intended to help); and futility (problems are so huge that nothing can be done about them). That certainly describes the current debates about global warming, illegal drugs and countless other topics. With luck, Mr Hirschman’s exit will not silence his voice.

  74. I did not act alone, but rather as part of a culture of climate denial among activists, who are already plagued by a tendency to see our work as separate issues vying for attention. The Armageddon Complex tells us that climate activism is about some far-off date, not about the pressing and time-sensitive needs that people around us experience in their day-to-day struggles. It pounds into us the idea that the crisis is more titanic than any other, so if we’re going to do anything about it, we have to do everything. Most of us won’t put off the pressing needs of our families and communities for something we abstractly understand is going to happen later, and most of us aren’t willing to drop the other pieces of our lives and our movement to do everything, because we already feel like we’re doing everything and barely scraping by as it is. So we deny.

  75. The report, entitled “A New Agenda on Climate Change: Facing up to Stealth Denial and Winding down on Fossil Fuels,” indicated that while less than 20 percent of the approximately 2,000 people surveyed are “unconvinced” that climate change is happening, nearly 64 percent of people say they acknowledge the reality of anthropogenic climate change yet do not feel a personal responsibility to address it.

    The findings have the report’s authors convinced that the best way to combat climate change is not to focus on the minority of people who deny the reality of human-caused climate change despite a 97 percent consensus among climate scientists, but rather to hone in on those who wrongfully believe that there is nothing they can do to change it.

    “Those who deny the reality of anthropogenic climate change are not at all helpful, but at least they are consistent,” Jonathan Rowson, one of the authors of the report, wrote on the organization’s blog on Tuesday. “One corollary of facing up to stealth denial is that we should turn more of our attention instead to mobilizing those who, like the author of this report, fully accept the moral imperative to act, but continue to live as though it were not there.”

    Of that 64 percent — which the report calls “stealth deniers” — 47 percent were “emotional” deniers, meaning they don’t feel personally uneasy about climate change. Twenty-six percent of those people were “personal” deniers who believe their own daily actions are not part of the problem — and 65 percent are “practical” deniers, believing that there is literally “nothing I can do personally that will have any significant effect on limiting climate change.” Only a small group of the total amount of people surveyed, 14.5 percent, said they lived in a way that they felt was consistent with their understanding of the problem.

  76. We characterise ‘stealth denial’ in terms of those who accept the reality of man-made climate change but who agree with at least one of the following narratives (which are not mutually exclusive). Emotional Denial: “I don’t feel uneasy about climate change”; Personal Denial: “My daily actions are not part of the climate change problem”; Practical Denial: “There is nothing I can do personally that will have any significant effect on limiting climate change”

    http://www.theguardian.com/sustainable-business/behavioural-insights/climate-change-denial-behaviour-change

  77. New U.S. ambassador to Canada Kelly Craft says she believes ‘both sides’ of climate science

    Former Republican fundraiser is the 1st woman to serve as top U.S. diplomat in Ottawa

    The new U.S. ambassador to Canada says that when it comes to climate change she believes in “both sides of the science.”

    Kelly Craft, who took up her position Monday, told the CBC’s Rosemary Barton she appreciated all of the scientific evidence on climate change.

    “I think that both sides have their own results, from their studies, and I appreciate and I respect both sides of the science,” Craft told Barton.

    She also said that while U.S. President Donald Trump’s approach to climate change is different from the government of Canada’s, both the U.S. and Canada have the same goal: to “better our environment and to maintain the environment.”

    Craft said the U.S. can still fight climate change even though her country has signalled it will leave the Paris climate change accord.

  78. The third reason for the conservative elite’s more nuanced view of environmental policy relates to public opinion—and is depressing. Having been subject to a decades-long misinformation campaign against climate science, conservative voters are so reliably sceptical they need no further priming. Until the mid-1990s Republicans and Democrats were similarly worried about global warming. But after a deluge of bogus science and conspiracy theories swamped right-wing media, their opinions diverged: 66% of Democrats now say they are very concerned about it; only 18% of Republicans say the same. This has transformed the issue from one of America’s least partisan, to one of the most, such that the remaining 82% of Republican voters appear resistant to reasoned argument on it: climate change is something lefties worry about, so they by definition do not. That, in turn, makes life easier for opportunists such as Mr Pruitt. Where they once risked being found out by their voters, they can now make whatever reality based compromises they like, so long as they keep enraging the other side. And Mr Pruitt is expert at that.

    These forces help explain Mr Pruitt’s recent pragmatism, and suggest his actions will be more moderate than his sceptical rhetoric suggests. Even so, he is weakening or scrapping most of the protections he can, while also running the agency down. By one projection, the EPA will cut its 15,000 strong staff in half by 2020. An EPA official describes this approach as “salting the Earth, not burning the place down.” That is hardly reassuring, considering the environmental vandalism Mr Pruitt is doing, and the vandalism to America’s Enlightenment traditions he represents.

  79. In 2015, after the Supreme Court had affirmed the constitutionality of same-sex marriage, Jeremy Frimer of the University of Winnipeg asked 200 randomly selected Americans to take part in a simple experiment. Having established who was for and who was against gay marriage, he told them they had been entered into a lottery to win $10. He then asked those in favour to read and respond to eight arguments against their view. They were allowed instead to read and reply to eight arguments in favour, but in that case, the value of their lottery prize would be reduced to $7. Those against same-sex marriage were offered the same trade-off: they could keep their $10 ticket if they read eight arguments in favour, but the prize was lower if they chose to read arguments against. Almost two-thirds (on both sides) gave up the chance of winning a little extra to avoid being exposed to the other point of view. People look for, remember and in this case are willing to forgo money for information that confirms their pre-existing beliefs.

    On November 27th President Trump dismissed a 1,650-page National Climate Assessment in which 13 federal agencies gave warning about the costs and dangers of global warming. “People like myself, we have very high levels of intelligence,” he told the Washington Post, modestly, “but we’re not necessarily such believers.” The president is speaking for his core supporters, too. According to the Pew Research Centre, only 15% of conservative Republicans trust scientists to give full and accurate information about the causes of climate change, compared with 70% of liberal Democrats. Most explanations for the extent of climate denial in America focus on the political influence and campaign contributions of energy companies. But as Mr Frimer’s experiment suggests, psychological explanations also suggest that people are willing to dismiss or deny facts and opinions that run counter to their beliefs.

    Such behaviour might seem short-sighted and self-defeating. But in a book of 2017, “The Enigma of Reason”, two cognitive scientists, Hugo Mercier and Dan Sperber, argue that reasoning did not evolve “to help individuals achieve greater knowledge and make better decisions”. Rather, they say, it evolved to improve the ability of ancestral hunter-gatherers to co-operate in small groups. As they put it: “What reason does…is help us justify our beliefs and actions to others…and evaluate the justifications and arguments that others address to us.” In other words, a lot of reasoning is devoted to affirming your group’s identity and your position within it.

    https://www.economist.com/united-states/2018/12/08/what-psychology-experiments-tell-you-about-why-people-deny-facts

  80. People with extreme anti-science views know the least, but think they know the most: study

    People often suffer from an ‘illusion of knowledge,’ write the authors of a new study that finds that people who hold the most extreme views about genetically modified foods know the least

    “The less people know,” the authors conclude, “the more opposed they are to the scientific consensus.”

    “Science communicators have made concerted efforts to educate the public with an eye to bringing their attitudes in line with the experts,”

    But people with an inflated sense of what they actually know — and most in need of education — are also the ones least likely to be open to new information.

    “This suggests that a pre-requisite to changing people’s views through education may be getting them to first appreciate the gaps in their knowledge,” the authors write.

    The problem is similar to the Dunning-Kruger effect: The less competent a person is at something, the smarter they think they are.

  81. In 2008 and 2009, the American Psychological Association put together a task force to examine the relationship between psychology and climate change. It found that, although people said that climate change was important, they did not “feel a sense of urgency.” The task force identified several mental barriers that contributed to this blasé stance. People were uncertain about climate change, mistrustful of the science, or denied that it was related to human activity. They tended to minimize the risks and believe that there was plenty of time to make changes before the real impacts were felt. Just ten years later, these attitudes about climate feel like ancient relics. But two key factors, which the task force identified as keeping people from taking action, have stood the test of time: one was habit, and the other was lack of control. “Ingrained behaviors are extremely resistant to permanent change,” the group stated. “People believe their actions would be too small to make a difference and choose to do nothing.”

    https://www.newyorker.com/science/elements/the-other-kind-of-climate-denialism

  82. Countering Climate Science Denial and Communicating Scientific Consensus

    John Cook

    Scientific agreement on climate change has strengthened over the past few decades, with around 97% of publishing climate scientists agreeing that human activity is causing global warming. While scientific understanding has strengthened, a small but persistent proportion of the public actively opposes the mainstream scientific position. A number of factors contribute to this rejection of scientific evidence, with political ideology playing a key role. Conservative think tanks, supported with funding from vested interests, have been and continue to be a prolific source of misinformation about climate change. A major strategy by opponents of climate mitigation policies has been to cast doubt on the level of scientific agreement on climate change, contributing to the gap between public perception of scientific agreement and the 97% expert consensus. This “consensus gap” decreases public support for mitigation policies, demonstrating that misconceptions can have significant societal consequences. While scientists need to communicate the consensus, they also need to be aware of the fact that misinformation can interfere with the communication of accurate scientific information. As a consequence, neutralizing the influence of misinformation is necessary. Two approaches to neutralize misinformation involve refuting myths after they have been received by recipients (debunking) or preemptively inoculating people before they receive misinformation (prebunking). Research indicates preemptive refutation or “prebunking” is more effective than debunking in reducing the influence of misinformation. Guidelines to practically implement responses (both preemptive and reactive) can be found in educational research, cognitive psychology, and a branch of psychological research known as inoculation theory. Synthesizing these separate lines of research yields a coherent set of recommendations for educators and communicators. Clearly communicating scientific concepts, such as the scientific consensus, is important, but scientific explanations should be coupled with inoculating explanations of how that science can be distorted.

  83. The psychology of denial concerning climate mitigation measures: evidence from Swiss focus groups

    Various studies of public opinion regarding the causes and consequences of climate change reveal both a deep reservoir of concern, yet also a muddle over causes, consequences and appropriate policy measures for mitigation. The technique adopted here, namely integrated assessment (IA) focus groups, in which groups of randomly selected individuals in Switzerland looked at models of possible consequences of climate change and questioned specialists as to their accuracy and meaning, revealed a rich assembly of reactions. Respondents were alarmed about the consequences of high-energy futures, and mollified by images of low-energy futures. Yet they also erected a series of psychological barriers to justify why they should not act either individually or through collective institutions to mitigate climate change. From the viewpoint of changing their lifestyles of material comfort and high-energy dependence, they regarded the consequences of possible behavioural shift arising from the need to meet mitigation measures as more daunting. To overcome the dissonance created in their minds they created a number of socio-psychological denial mechanisms. Such mechanisms heightened the costs of shifting away from comfortable lifestyles, set blame on the inaction of others, including governments, and emphasised doubts regarding the immediacy of personal action when the effects of climate change seemed uncertain and far away. These findings suggest that more attention needs to be given to the social and psychological motivations as to why individuals erect barriers to their personal commitment to climate change mitigation, even when professing anxiety over climate futures. Prolonged and progressive packages of information tailored to cultural models or organised belief patterns, coupled to greater community based policy incentives may help to widen the basis of personal and moral responsibility.

  84. The dragons of inaction: Psychological barriers that limit climate change mitigation and adaptation.
    By Gifford, Robert
    American Psychologist, Vol 66(4), May-Jun 2011, 290-302

    Most people think climate change and sustainability are important problems, but too few global citizens engaged in high-greenhouse-gas-emitting behavior are engaged in enough mitigating behavior to stem the increasing flow of greenhouse gases and other environmental problems. Why is that? Structural barriers such as a climate-averse infrastructure are part of the answer, but psychological barriers also impede behavioral choices that would facilitate mitigation, adaptation, and environmental sustainability. Although many individuals are engaged in some ameliorative action, most could do more, but they are hindered by seven categories of psychological barriers, or “dragons of inaction”: limited cognition about the problem, ideological worldviews that tend to preclude pro-environmental attitudes and behavior, comparisons with key other people, sunk costs and behavioral momentum, discredence toward experts and authorities, perceived risks of change, and positive but inadequate behavior change. Structural barriers must be removed wherever possible, but this is unlikely to be sufficient. Psychologists must work with other scientists, technical experts, and policymakers to help citizens overcome these psychological barriers.

  85. Information regarding the potentially severe and arbitrary effects of global warming should constitute a significant threat to belief in a just world, and discrediting or denying global warming’s existence could serve as a means of resolving the resulting threat. Many dire messages aimed at stopping global warming make salient the impending chaos and unpredictable catastrophe that global warming will bring with it. Moreover, these messages often emphasize the harm that will be done to children and future generations who have done nothing themselves to cause global warming. Such messages contradict the belief that the world is predictable and fair by suggesting that good people will suffer and that the innocent will be the primary victims. Because these messages contradict just-world beliefs, individuals who most strongly hold such beliefs should be the most threatened.

    https://journals.sagepub.com/doi/pdf/10.1177/0956797610391911

  86. However, this created a dilemma for propertarians. Having rejected both state- and market-led solutions to climate change, their only remaining option was to deny that the problem existed at all. This required tremendous intellectual dishonesty, supplied in large measure by hired guns who had won their spurs in earlier fights over passive smoking and the ozone layer. This didn’t trouble the libertarian ground troops, who clung tenaciously to their baseless self-image as the smartest guys in the room. Lacking any scientific, economic, or statistical knowledge to back up their opinions, they seized upon the innumerable talking points that the denial industry churned out.

    https://www.jacobinmag.com/2020/01/libertarianism-climate-change-environment-private-property

  87. You say that despite a public scare based on the flawed WHI study, hormone therapy has the potential to safely improve women’s lives. Yet in “Naked Statistics”, written by Charles Wheelan in 2013, the WHI study is presented as a rigorous controlled experiment. He quotes a statement that the death toll from unnecessarily prescribed ht is in the “tens of thousands”. It would appear that the evidence is on The Economist’s side. Yet what is a non-specialist to do? Despite reaching opposite conclusions, both The Economist and Mr Wheelan write clearly and with the confidence of experts. Unfortunately, that leaves the rest of us ever warier.

    Dan Phillips
    New York

  88. Discourses of climate delay

    ‘Discourses of climate delay’ pervade current debates on climate action. These discourses accept the existence of climate change, but justify inaction or inadequate efforts. In contemporary discussions on what actions should be taken, by whom and how fast, proponents of climate delay would argue for minimal action or action taken by others. They focus attention on the negative social effects of climate policies and raise doubt that mitigation is possible. Here, we outline the common features of climate delay discourses and provide a guide to identifying them.

  89. But the deniers are not all the same. They tend to fit into one of four different categories: the shill, the grifter, the egomaniac and the ideological fool.

    The shill is the easiest to understand. He, and it almost always is he, is paid by vested interests to emit clouds of confusion about the science or economics of climate action. This uncertainty creates a smokescreen behind which polluters can lobby against measures that cut their profits.

    A sadder case is that of the grifters. They have found themselves earning a living by grinding out contrarian articles for rightwing media outlets. Do they actually believe the guff they write? It doesn’t matter: they just warm their hands on the outrage, count the clicks and wait for the pay cheque.

    The egomaniacs are also tragic figures. They are disappointed, frustrated people whose careers have stalled and who can’t understand why the world refuses to give full reverence to their brilliance. They are desperate for recognition, and, when it stubbornly refuses to arrive, they are drawn to make increasingly extreme pronouncements, in the hope of finally being proved a dogma-busting, 21st-century Galileo.

    The ideological fool is the fourth type of climate denier, and they can be intelligent. But they are utterly blinded by their inane, no-limits version of the free-market creed. The climate emergency requires coordinated global action, they observe, and that looks horribly like communism in disguise.

  90. But there is another form of denial, what the philosopher Quassim Cassam calls “behavioural or practical denialism”. This is the mindset that accepts the science marshalled by the IPCC – it hears the alarm bell ringing – but still does not change its behaviour. It can operate at the level of governments: note the White House official who on Wednesday urged global oil producers to open up the taps and increase production, so that hard-pressed US motorists can buy gasoline more cheaply. And it lives in individuals, too, in the fatalism that says one person can do nothing to halt a planetary emergency, so you might as well shrug and move on. Which is “to act in the same way as if you were a climate change denier,” says Cassam. “The practical upshot is the same.”

    Whether it’s Covid or climate, there is a common defect at work here. It is wilful blindness, a deliberate closing of the eyes to a reality that is too hard to bear – and it afflicts far more than a hardcore of noisy sceptics and protesters. A US poll this week found that a summer of heatwaves, flooding and wildfires – evidence that the planet is both burning and drowning – has barely shifted attitudes to the climate issue. Many, even most, are looking the other way.

    https://www.theguardian.com/commentisfree/2021/aug/13/denial-anti-vaxxers-climate-sceptics

Leave a Reply

Your email address will not be published. Required fields are marked *